Вы находитесь на странице: 1из 15

5/25/13 Exam Reports

testfunda.com/LMS/Student/NewReports.aspx 1/15
1.
1 Marks
R is a positive number. It is multiplied by 8 and then squared. The square is now divided by 4 and the square root is taken. The result of the
square root is Q. What is the value of Q?
[SNAP 2010]
1) 3R
2) 4R
3) 7R
4) 9R
Solution:


Q = 4R
Hence, option 2.
2.
1 Marks
A number G236G0 can be divided by 36 if G is:
[SNAP 2011]
1) 8
2) 6
3) 1
4) More than one values are possible.
Solution:
G236G0 is divisible by 36
Hence, G236G0 is divisible by 9 and G is an even number.
Hence, G + 2 + 3 + 6 + G + 0 = 2G + 11 is divisible by 9.
Only G = 8 satisfies the given condition.
Hence, option 1.
3.
1 Marks
The total number of Natural numbers that lie between 10 and 300 and are divisible by 9 is
[SNAP 2011]
1) 32
2) 30
3) 33
4) 34
Solution:
Smallest and largest numbers, in the given region, which are divisible by 9 are 18 and 297 respectively.
Hence, the required number = (297 18)/9 + 1 = 31 + 1 = 32
Collapse All
Section I
Reports for
Numbers - Past
Other MBA Test
Questions 2
Overview Solution Key Unattempted Questions Report

5/25/13 Exam Reports
testfunda.com/LMS/Student/NewReports.aspx 2/15
Hence, option 1.
4.
1 Marks
The average of 4 distinct prime numbers a, b, c, d is 35, where a < b < c < d. a and d are equidistant from 36 and b and c are equidistant
from 34 and a, b are equidistant from 30 and c and d are equidistant from 40. The difference between a and d is:
[SNAP 2011]
1) 30
2) 14
3) 21
4) Cannot be determined
Solution:
a and d are equidistant from 36.
This implies that average of a and d is 36.
Hence, a + d = 72
Similarly, we have,
b + c = 68, a + b = 60 and c + d = 80
Now, a < b and a + b = 60
Hence, a < 30 and b > 30
Similarly,
b < 34 and c > 34
And c < 40 and d > 40
Now, only primes which satisfy this conditions are,
a = 29, b = 31, c = 37 and d = 43
Hence, d a = 43 29 = 14
Hence, option 2.
5.
1 Marks
Four digits of the number 29138576 are omitted so that the result is as large as possible. The largest omitted digit is.
[XAT 2008]
1) 9
2) 8
3) 7
4) 6
5) 5
Solution:
We have to omit four digits in such a manner that we get the largest possible number as the result.
We will omit 2, 1, 3 and 5 and the result will be 9876.
The largest omitted digit will be 5.
Hence, option 5.
6.
1 Marks
F(x) is a fourth order polynomial with integer coefficients and no common factor. The roots of F(x) are 2, 1, 1, 2. If p is a prime number
greater than 97, then the largest integer that divides F(p) for all values of p is:
[XAT 2009]
1) 72
2) 120
3) 240
5/25/13 Exam Reports
testfunda.com/LMS/Student/NewReports.aspx 3/15
4) 360
5) None of the above.
Solution:
Roots of F(x) are 2, 1, 1, 2, we have,
So we have, F(x) = (x + 2)(x + 1)(x 1)(x 2)
F(p) = (p + 2)(p + 1)(p 1)(p 2) where p is the prime number > 97
Let k be the largest integer which can divide F(p).
In (p + 2), (p + 1), (p 1) and (p 2), there will two even numbers and two odd numbers as p itself is an odd number.
Now, out of the two even numbers, one surely will be divisible by 4 and other by 2. So these two numbers are collectively divisible by 8.
Now, (p + 2), (p + 1), p, (p 1) and (p 2) are five consecutive integers. So either (p + 1) and (p 2) or (p + 2) and (p 1), each will be
divisible by 3. So either of these two pairs are divisible by 9.
Also, being 5 consecutive numbers, it must contain one number which is multiple by 5. So this number will be divisible by 5.
So collectively F(p) will be divisible by 8 9 5 = 360
Hence, option 4.
Group Question

Answer the following questions based on the information given below.
In the diagram below, the seven letters correspond to seven unique digits chosen from 0 to 9. The relationship among the digits is such that:
P.Q.R = X.Y.Z = Q.A.Y
[XAT 2009]
7.
1 Marks
The value of A is:
1) 0
2) 2
3) 3
4) 6
5) None of the above.
Solution:
After trial and error, we get, 8 9 1 = 4 2 9 = 6 4 3 = 72
From this above table, we get that, A is equal to 2.
Hence, option 2.
5/25/13 Exam Reports
testfunda.com/LMS/Student/NewReports.aspx 4/15
8.
1 Marks
The sum of the digits which are not used is:
1) 8
2) 10
3) 14
4) 15
5) None of the above
Solution:
Digits which are not used are 0, 5, 7.
Sum of these digits = 5 + 7 + 0 = 12
Hence, option 5.
9.
1 Marks
Company BELIANCE hosted a party for 8 members of Company AXIAL. In the party no member of AXIAL had interacted with more than
three members of BELIANCE. Out of all the members of BELIANCE, three members - each interacted with four members of AXIAL and the
remaining members - each interacted with two members of AXIAL. The greatest possible number of company BELIANCE in the party is
[XAT 2009]
1) 9
2) 10
3) 11
4) 12
5) None of the above.
Solution:
There are 8 members in AXIAL, and each of them interacts with a maximum of 3 members from BELIANCE.
A maximum of 8 3 = 24 interactions are possible.
Now, assume that there are X members in BELIANCE. Since 3 members of BELIANCE each interact with 4 members of AXIAL, 12
interactions occur in this process.
24 12 = 12 interactions remain; that is, 12 is the maximum number of interactions that could occur among the remaining members.
The remaining members in BELIANCE is (X 3) and each of them interact with 2 members of AXIAL, therefore interactions will be
maximum when (X 3) 2 = 12, that is when X = 9
Hence, option 1.
10.
1 Marks
Let X be a four digit number with exactly three consecutive digits being same and is a multiple of 9. How many such Xs are possible?
[XAT 2009]
1) 12
2) 16
3) 19
4) 21
5) None of the above
Solution:
X is a four-digit number that is divisible by 9. This means that the sum of its digits should add up to a multiple of 9.
It is also given that exactly 3 consecutive digits are the same. Hence, two possible patterns of the number arise: YYYZ and ZYYY
Case 1: The first three digits are the same, and the fourth one is different; i.e. YYYZ
Here, we will substitute Y by values from 1 to 9 (not 0, since Y is the first digit), and see how many values Z can take such that YYYZ will
be a multiple of 9.
For example, when Y is 1, then the first three digits add up to 3. Since (Y + Y + Y + Z) must be some multiple of 9, hence Z can only be 6
(i.e. the number will be 1116). Similarly, when Y is 3, then the first three digits add up to 9. Hence, the fourth digit could be either 0 or 9 (i.e.
the numbers formed will be 3330 and 3339 both multiples of 9).
5/25/13 Exam Reports
testfunda.com/LMS/Student/NewReports.aspx 5/15
the numbers formed will be 3330 and 3339 both multiples of 9).

* When Y is 9, then the first three digits add up to 27. Hence, the fourth digit could be either 0 or 9. However, if the fourth digit was 9, then
the number formed would be 9999, which doesnt satisfy the criteria that exactly THREE consecutive digits should be the same.
This case leads to 1 + 1 + 2 + 1 + 1 + 2 + 1 + 1 + 1 = 11 possibilities
Case 2: The last three digits are the same, and the first one is different; i.e. ZYYY
Again, we substitute Y by values from 0 to 9 (this time Z is the first digit, so Y can be 0), and see how many values of Z can satisfy the
criteria that ZYYY is a multiple of 9.
For example, when Y is 1, the last three digits add up to 3. Hence, Z can take the value 6. When Y is 3, the sum of the last three digits is
9; so Z will be 9 (Z, being the first digit, cannot be 0).

5/25/13 Exam Reports
testfunda.com/LMS/Student/NewReports.aspx 6/15

This case leads to 1 9 = 9 possibilities.
A total of 11 + 9 = 20 four-digit numbers are possible that is a multiple of 9, with exactly 3 consecutive digits being same.
Hence, option 5.
11.
1 Marks
Let X be a four-digit positive integer such that the unit digit of X is prime and the product of all digits of X is also prime. How many such
integers are possible?
[XAT 2010]
1) 4
2) 8
3) 12
4) 24
5) None of the above
Solution:
Unit digit of x can be 2, 3, 5, or 7.
As the product of all digits of x is prime, x can only be 1112, 1113, 1115 or 1117.
Hence, option 1.
12.
1 Marks
a, b, c, d and e are integers such that 1 a < b < c < d < e. If a, b, c, d and e are geometric progression and
lcm(m, n) is the least common multiple of m and n, then the maximum value of
[XAT 2010]
1) 1
2)
3)
4)
5) None of the above
Solution:
Let a be the first term and r the common ratio of the geometric progression.
b = ar, c = ar
2
, d = ar
3
and e = ar
4


5/25/13 Exam Reports
testfunda.com/LMS/Student/NewReports.aspx 7/15



To get the maximum value of the above expression, denominator has to be minimum.
So, we take a = 1 and r = 2

For any other value of r, p would be less than this. So 15/16 is the maximum value.
Hence, option 2.
13.
1 Marks
A chocolate dealer has to send chocolates of three brands to a shopkeeper. All the brands are packed in boxes of same size. The number
of boxes to be sent is 96 of brand A, 240 of brand B and 336 of brand C. These boxes are to be packed in cartons of same size containing
equal number of boxes. Each carton should contain boxes of same brand of chocolates. What could be the minimum number of cartons
that the dealer has to send?
[XAT 2010]
1) 20
2) 14
3) 42
4) 38
5) 16
Solution:
Number of boxes of brand A, B and C are 96, 240 and 336 respectively.
Each carton should contain boxes of same brand and also an equal number of boxes.
This can happen only when each carton contains a number of boxes that is a factor of number of boxes of each brand i.e. a factor of 96, 240
and336.
To get minimum number of cartons, number of boxes in each carton should be maximum.
So, we should take the highest common factor of 96, 240 and 336.
96 = 2
5
3
240 = 2
4
3 5
and 336 = 2
4
3 7
So, the highest common factor = 2
4
3 = 48



Total number of cartons = 2 + 5 + 7 = 14
Hence, option 2.
14.
1 Marks

where [x] is defined as integral part of x and f is a faction, then x(1 f) equals-
5/25/13 Exam Reports
testfunda.com/LMS/Student/NewReports.aspx 8/15
where [x] is defined as integral part of x and f is a faction, then x(1 f) equals-
[XAT 2011]
1) 1
2) Less than 1
3) More than 1
4) Between 1 and 2
5) None of the above
Solution:

Let

Now,

Also,



x + y = 2(k) = even
Now,


0 < y < 1 (ii)
Also, x = [x] + f, 0 < f < 1 (iii)
[x] + f + y is even
As [x] is an integer, f + y is an integer.
From (ii) and (iii)
0 < f + y < 2
f + y = 1
Now, x (1 f) = xy
But from (i), xy = 1
x(1 f) = 1
Hence, option 1.
15.
1 Marks
Let a
n
= 1 1 1 1 1 1 1.. 1, where 1 occurs n number of times. Then,
i. a
741
is not a prime.
ii. a
534
is not a prime.
iii. a
123
is not a prime.
iv. a
77
is not a prime.
[XAT 2011]
1) (i) is correct.

5/25/13 Exam Reports
testfunda.com/LMS/Student/NewReports.aspx 9/15
2) (i) and (ii) are correct.
3) (ii) and (iii) are correct.
4) All of them are correct.
5) None of them is correct.
Solution:
a
741
has 1 written 741 times.
The sum of digits of a
741
is divisible by 3.
a
741
is divisible by 3.
a
534
and a
123
are also not prime by the same logic.
(i), (ii), and (iii) are correct.
Option (4) can be safely assumed to be the correct option.
Hence, option 4.
Group Question

Answer the following questions based on the information given below.
From a group of 545 contenders, a party has to select a leader. Even after holding a series of meetings, the politicians and the general body
failed to reach a consensus. It was then proposed that all 545 contenders be given a number from 1 to 545. Then they will be asked to stand on a
podium in a circular arrangement, and counting would start from the contender numbered 1. The counting would be done in a clockwise fashion.
The rule is that every alternate contender would be asked to step down as the counting continued, with the circle getting smaller and smaller, till
only one person remains standing. Therefore the first person to be eliminated would be the contender numbered 2.
[XAT 2011]
16.
1 Marks
Which position should a contender choose if he has to be the leader?
1) 3
2) 67
3) 195
4) 323
5) 451
Solution:
Let, f(n) represent the position of winner when n persons are standing in a circle.
f (n) = 2l + 1
where, n = 2
m
+ l and 0 l < 2
m
Now, n = 545
n = 512 + 33
n = 2
9
+ 33
l = 33
f (545) = 2 33 + 1 = 67
Hence, option 2.
Note: This question is based on the Josephus Flavius problem.
17.
1 Marks
One of the contending politicians, Mr. Chanaya, was quite proficient in calculations and could correctly figure out the exact position.
He was the last person remaining in the circle. Sensing foul play the politicians decided to repeat the game. However, this time,
instead of removing every alternate person, they agreed on removing every 300th person from the circle. All other rules were kept
intact. Mr. Chanaya did some quick calculations and found that for a group of 542 people the right position to become a leader would
be 437. What is the right position for the whole group of 545 as per the modified rule?
1) 3
2) 194
5/25/13 Exam Reports
testfunda.com/LMS/Student/NewReports.aspx 10/15
2) 194
3) 249
4) 437
5) 543
Solution:
Let f (n, k) represent the position of a winner when there are n people out of which every k
th
person is eliminated.
We have,
f (n, k) = (f (n 1, k) + k) mod n
Now f (542, 300) = 437
Hence,
f (543, 300) = (437 + 300) mod 543 = 194
f (544, 300) = (194 + 300) mod 544 = 494
f (545, 300) = (494 + 300) mod 545 = 249
A contender at 249
th
position will win the election.
Hence, option 3.
Note: This question is based on the Josephus Flavius problem.
18.
1 Marks
Little Pika who is five and half years old has just learnt addition. However, he does not know how to carry. For example, he can add 14 and
5, but he does not know how to add 14 and 7. How many pairs of consecutive integers between 1000 and 2000 (both 1000 and 2000
included) can Little Pika add?
[XAT 2011]
1) 150
2) 155
3) 156
4) 258
5) None of the above
Solution:
Let a, b and c be three digits belonging to the set {0, 1, 2, 3, 4}
The first number of the two consecutive numbers can be of following types

Hence total number of pairs = 125 + 25 + 5 + 1 = 156
Hence, option 3.
19.
1 Marks
The micromanometer in a certain factory can measure the pressure inside the gas chamber from 1 unit to 999999 units. Lately this
instrument has not been working properly. The problem with the instrument is that it always skips the digit 5 and moves directly from 4 to 6.
What is the actual pressure inside the gas chamber if the micromanometer displays 0030l6?
[XAT 2011]
5/25/13 Exam Reports
testfunda.com/LMS/Student/NewReports.aspx 11/15
1) 2201
2) 2202
3) 2600
4) 2960
5) None of the above options
Solution:
Skipping the digit 5 converts the counting base system to 9 from 10.
Actual values of digits greater than 5 will change i.e. face value of 6 will be 5, 7 will be 6, and so on.
Hence, 3016 displayed by micromanometer is actually 3015 in base 9
Now, (3015)
9
= 3 9
3
+ 0 9
2
+ 1 9 + 5 = 2201
Hence, option 1.
20.
1 Marks
Three Vice Presidents (VP) regularly visit the plant on different days. Due to labour unrest, VP (HR) regularly visits the plant after a gap of 2
days. VP (Operations) regularly visits the plant after a gap of 3 days. VP (Sales) regularly visits the plant after a gap of 5 days. The VPs do
not deviate from their individual schedules. CEO of the company meets the VPs when all the three VPs come to the plant together. CEO is
one leave from January 5
th
to January 28
th
, 2012. Last time CEO met the VPs on January 3, 2012. When is the next time CEO will meet all
the VPs?
[XAT 2012]
1) February 6, 2012
2) February 7, 2012
3) February 8, 2012
4) February 9, 2012
5) None of the above
Solution:
VP (HR) visits on every third day, VP (Operations) visits on every fourth day and VP (Sales) visits on every sixth day.
Hence, all of them will visit together on every twelfth day.
Now, all VPs visited together on January 3, 2012.
Hence, they will visit on, 15
th
January, 27
th
January, 8
th
February and so on.
Hence, option 3.
21.
1 Marks
Lionel and Ronaldo had a discussion on the ages of Joses sons. Ronaldo made following statements about Joses sons:
i. Jose has three sons.
ii. The sum of the ages of Joses sons is 13.
iii. The product of the ages of the sons is the same as the age of Lionel.
iv. Joses eldest son, Zizou weighs 32 kilos.
v. The sum of the ages of the younger sons of Jose is 4.
vi. Jose has fathered a twin.
vii. Jose is not the father of a triplet.
viii. The LCM of the ages of Joses sons is more than the sum of their ages.
Which of the following combination gives information sufficient to determine the ages of Joses sons?
[XAT 2012]
1) i, ii, iii and iv
2) i, ii, iv and vi
3) i, ii, iii and v
4) i, ii, v and vii
5) i, ii, v and vi
Solution:
Consider each option separately,
5/25/13 Exam Reports
testfunda.com/LMS/Student/NewReports.aspx 12/15
Consider option A:
In this case, we dont know the Lionels age.
Hence, the only conclusion can be derived is;
Jose has 3 sons and sum of their age is 13.
Consider option B:
In this case also, we only know that Jose has 3 sons and sum of their ages is 13 and two of them are twin.
But it still doesnt provide adequate information to calculate age of each son.
Consider option C:
As, age of Lionel is not known, we only know that the sum of ages of Joses three son is 13 out of which sum of the age of the younger two
brother is 4.
By this we can calculate age of the Joses eldest son, but we cannot age of the remaining two children.
Consider option D:
In this case, statement 7 is redundant, as by statement 2 only we can guess that Jose is not the father of a triplet.
Hence this statement doesnt provide adequate information to guess the ages of all the three children.
Consider option E:
By, i, ii and iii, we can conclude that the age of the Joses eldest son is 9 and sum of the other two children is 4.
Now, by statement vi, Jose has fathered twins, we can easily conclude that the younger two children are twins.
Hence, their ages are 2 and 2 respectively.
Hence, E is sufficient to answer.
Hence, option 5.
22.
1 Marks
Z is the product of first 31 natural numbers. If X = Z + 1, then the numbers of primes among X + 1, X + 2, ..., X + 29, X + 30 is
[IIFT 2012]
1) 30
2) 2
3) Cannot be determined
4) None of the above
Solution:
Z = 31!
Z is divisible by all numbers less than 32.
X = 31! + 1
X + 1 = 31! + 2, will be divisible by 2,
X + 2 = 31! + 3 will be divisible by 3,
X + 3 = 31! + 4 will be divisible by 4 and so on.
Hence none of the numbers will be prime.
Hence, option 4.
23.
1 Marks
Consider the expression: (xxx)
b
x
3
, where b is the base, and x is any digit of base b. Find the value of b.
[XAT 2013]
1) 5
2) 6
3) 7
5/25/13 Exam Reports
testfunda.com/LMS/Student/NewReports.aspx 13/15
3) 7
4) 8
5) None of the above
Solution:
Since x is any digit in the base, let us consider x = 1.
Substituting the value of b for any of the options i.e.5, 6, 7, 8, we get
(111)
5
1
3
(111)
6
1
3
(111)
7
1
3
(111)
8
1
3
Hence, none of the above is true.
Hence, option 5.
24.
1 Marks
If k is an integer and 0.0010101 10k is greater than 1000, what is the least possible value of k?
[IIFT 2012]
1) 4
2) 5
3) 6
4) 7
Solution:
We need 0.0010101 10
k
> 1000
1010.1 10
6
10
k
> 1000
1010.1 10
6 + k
> 1000
1010.1 > 1000, 10
6 + k
= 1
k 6 = 0
k = 6
Hence, option 3.
25.
1 Marks
The unit digit in the product of (8267)
153
(341)
72
is
[IIFT 2012]
1) 1
2) 2
3) 7
4) 9
Solution:
The units digit of (8267)
153
is same as unit digit of 7
153
.
Since cyclicity of 7 is 4 and the remainder obtained when 153 is divided by 4 is 1,
Units digit of (8267)
153
= units digit of 7
153
= units digit of 7
1
= 7
Similarly,
Units digit of (341)
72
is same as units digit of 1
72
= 1
Hence the units digit of the product = 7 1 = 7
5/25/13 Exam Reports
testfunda.com/LMS/Student/NewReports.aspx 14/15
Hence, option 3.
26.
1 Marks
How many whole numbers between 100 and 800 contain the digit 2?
[XAT 2013]
1) 200
2) 214
3) 220
4) 240
5) 248
Solution:
Between 100 and 199, there will be 19 numbers which contain 2. They are as follows:
102, 112, 120 129 (10 numbers), 132, 142, 152, 162, 172, 182, 192
Similar would be the case for 300 - 399, 400 - 499, 500 - 599 and 600 - 699.
For 200 299, all 100 numbers will have 2.
Total number of numbers = 19 6 + 100 = 114 + 100 = 214
Hence, option 2.
27.
1 Marks
A number is interesting if on adding the sum of the digits of the number and the product of the digits of the number, the result is equal to the
number. What fraction of numbers between 10 and 100 (both 10 and 100 included) is interesting?
[XAT 2013]
1) 0.1
2) 0.11
3) 0.16
4) 0.22
5) None of the above
Solution:
Let the numbers be of the form 10x + y.
10x + y = x + y + xy
9x = xy
y = 9
The numbers are 19, 29, 39, 49, 59, 69, 79, 89 and 99 - total of 9 numbers.
Now, the total number of numbers = 100 (10 1) = 91

Hence, option 5.
28.
1 Marks
Please read the following sentences carefully.
a. 103 and 7 are they only prime factors of 1000027
b.
c. If I travel one half of my journey at an average speed of x km/h, it will be impossible for me to attain an average speed of 2x km/h for
the entire journey.
[XAT 2013]
1) All the statement are correct
2) Only Statement II is correct
5/25/13 Exam Reports
testfunda.com/LMS/Student/NewReports.aspx 15/15
2) Only Statement II is correct
3) Only statement III is correct
4) Both statements I and II are correct
5) Both statement I and III are correct
Solution:
Statement I Prime factors of 1000027 are 7, 19, 73, 103. Hence, the statement is false.
Statement III It is true. Let us solve it taking an example.
E.g. Let s
1
= 30 kmph.
Average speed = 60 kmph

60s
1
+ 1800 = 60s
1
. This is mathematically incorrect.
Hence, the only option which satisfies the condition is 3
Hence, option 3.

Вам также может понравиться